Đến nội dung

Hình ảnh

Một bài về hàm lồi (Karamata và JenSen)

- - - - -

  • Please log in to reply
Chưa có bài trả lời

#1
minhhieu070298vn

minhhieu070298vn

    Trung sĩ

  • Thành viên
  • 177 Bài viết

 Cho x1,x2,....,xn thực, $x_{i}x_{j}\leq 4^{-\left | i-j \right |}(1\leq i\leq j\leq n).$

 CMR $x_{1}+x_{2}+...+x_{n} \leq \frac{5}{3}$


Bài viết đã được chỉnh sửa nội dung bởi minhhieu070298vn: 08-11-2015 - 20:10





1 người đang xem chủ đề

0 thành viên, 1 khách, 0 thành viên ẩn danh